LSAT and Law School Admissions Forum

Get expert LSAT preparation and law school admissions advice from PowerScore Test Preparation.

User avatar
 Dave Killoran
PowerScore Staff
  • PowerScore Staff
  • Posts: 5853
  • Joined: Mar 25, 2011
|
#90420
Complete Question Explanation
(The complete setup for this game can be found here: lsat/viewtopic.php?t=980)

The correct answer choice is (B)

If K occupies the aisle seat in row 3, then the fifth rule is activated and I must sit in row 1. However, because the GH block occupies rows 1 and 2 of the aisle seats, I must sit in the row 1 window seat, leaving L and M as the other two window seats:

G3-Q17-d1.png

Accordingly, answer choice (B) is not necessarily true and therefore correct.
You do not have the required permissions to view the files attached to this post.

Get the most out of your LSAT Prep Plus subscription.

Analyze and track your performance with our Testing and Analytics Package.